You are on page 1of 85

STATICS OF RIGID BODIES

Mechanics - defined as the


science which describes and predicts the condition of rest or motion of bodies under the action of
force
Rigid body - definite amount of matter, the parts of which are fixed in position relative to one
another.
Mass – invariant property of a body which measures its resistance to a change of motion
Force - action exerted by one body upon another.

TWO DIVISIONS OF FORCE SYSTEM


 Coplanar Force System - line of action are on the same plane.

Point of Application
OBJECT
Line of Action

Concurrent Coplanar Force System


Non-Concurrent Coplanar Force System Parallel Coplanar Force System
 Non-Coplanar Force System - line of action are not on the same plane.

z
z z

y
y y

x
Concurrent Non-Coplanar Force System x x
Non-Concurrent Non-Coplanar Force System Parallel Non-Coplanar Force System

CHARACTERISTICS OF A FORCE
1. Magnitude (e.g. 500N, 10 kN, 1000 lb,8 kip)
2. Position of its line of action (horizontal, diagonal)
3. Direction (rightward, leftward, upward, downward)

TYPES OF FORCE DISTRIBUTION


1. Concentrated Force

2. Uniformly Distributed Force (Rectangular Load)


3. Uniformly Varying Force (Triangular Load)

4. Trapezoidal Force

COMPONENTS OF FORCES
1. If angle is given:

Fy F

F
Fy
θy
θ
θx
Fxx
Fx
Fy = F Sinθx
Fy= F Cosθy
Fx = F Sinθy
Fx = F Cosθx

2. If slope is given:

Fy
Fy
F
c F
a Fy
b
Fxx
Fx

By Ratio and Proportion:


F Fy Fx
= =
c a b
b
Fx= F
c
a
Fy= F
c

SAMPLE PROBLEMS:
Problem 1: For the following forces, determine the horizontal and vertical components of the
force. Indicate whether it is positive or negative.

A=50kN a.) A = 50 kN
Ay
Ax = 50kN Cos 25°
Ax = 45.32kN
Ax = -50kNSin 25°
25° Ay = -21.13 kN
Axx

By B= 4000N
b.) B = 4000 N
Bx = -4000N Sin 70°
Bx = -3758.71 N
70°
Bx = 4000 N Cos 70°
Bxx By = 1368.08 N

Cxx c.) C = 18 kips


7
Cx = 18 kips ( )
3 7.62
Cx = 16.54 kips
7
3
Cy = -18 kips ( )
7.62
Cy C = 18 kips
Cy = -7.09 kips

Dy d.) D = 3200 lb
D = 3200lb
12
Dx = 3200 lb ( )
13
13 Dx = 2953.81 lb.
5
5
Dy = 3200 lb ( )
13
Dxx
Dy = 1230.77 lb.
x
E = 10 kN e.) E = 10 kN
Ex = 10 kN Cos 55°
75° Ex = 5.74 kN
Ex = -10kN Sin 55°
Problem 2: The vertical force F acts at a on the two membered
Ey = -8.19 frame,
kN determine the magnitude
at the component of F directed along the axes of AB and AC.
20°
Given:
F= 550 N
Required:
FAB, FAC
Solution:
USING SINE LAW:
F FAB FAC
= =
c a b
550 N FAB FAC FAC
= = 50°
sin 70° sin 60 ° sin 50 °
F = 550 N
70°
FAB =506.88 N 60°
FAB
FAC = 448.36 N

Problem 3: The force acting on the gear tooth is F= 20 lb. determine the component force acting
along line aa and line bb.

Given:
F 20 lb.
Required:
Faa, Fbb
Solution:
USING SINE LAW:
F Faa Fbb
= =
c a b
20 lb Faa Fbb 0 lb
= = =2
sin 40 ° sin 80 ° sin 60 ° F
°
80 °
Faa=30.64 lb 60
b
Fb 40
°
Fbb = 26.95 lb a
Fa

TRANSMISSIBILITY OF A FORCE
The point of application of a force can be transferred along its line of action without affecting the
other external forces
RESULTANT OF CONCURRENT COPLANAR FORCES
Two or more concurrent coplanar forces can be replaced by a single force with the same effect as
those of concurrent forces. The single force is called Resultant Force. In symbol,

R² = (ΣFx)² + (ΣFy)² ------ R = √(ΣFx)² + (ΣFy)²

where:
R = Resultant Force (N, kN, lb, kip)
ΣFx = Summation of forces along horizontal axis
ΣFy = Summation of forces along vertical axis

SAMPLE PROBLEMS:

Problem 1: Determine the magnitude and location of the resultant force for the concurrent force
system shown in the figure.

Req'd: R, θ

ΣFx = Ax - Bx + Cx + Dx + Ex - Fx
= 0 kN - 18cos30° kN + 20 kN + 16 kN
(2/√53) + 9 kN (4/8) -22sin65° kN
= -6.03 kN
ΣFy = Ay - By + Cy - Dy + Ey + Fy
= 24 kN - 18sin30° kN + 0 kN - 16 kN (7/√53) + 9 kN (√48/8) + 22cos65° kN
= 16.71 kN

R = √(ΣFx)² + (ΣFy)²
= √(-6.03 kN)² + (16.71 kN)²
= 17.98 kN

tan θ = ΣFx
ΣFy
θ = tan⁻¹ 16.71 kN
6.03 kN
θ = 68.36°

Problem 2: The force system shown in the figure has a resultant of 215 N pointing upwards
along the y-axis. Determine the values of F and θ to give the resultant.
Req'd: F, θ

ΣFx = Rx; Fcosθ - 250cos30° N - 500 N = 0


Fcosθ = 293.49 N eq. 1

ΣFy = Ry; Fsinθ - 250sin30° N = 215 N


Fsinθ = 340 N eq. 2

Equate eq. 1 and eq. 2:


Fsinθ = 340 N
Fcosθ 293.49 N

sinθ = 340 N
cosθ 293.49 N

tanθ = 340 N
293.49 N

θ = tan⁻¹ 340 N
293.49 N

θ = 49.20°

Substitute the θ to eq. 1:


Fcosθ = 293.49 N
Fcos49.20° = 293.49 N
F = 293.49 N
cos49.20°

F = 449.14 N

Problem 3: From the bracket shown in the figure loaded with three forces. Determine the values
of F₁ and θ so that the resultant force is directed to the positive x-axis and has a magnitude of 800
N.

Req'd: F₁, θ

ΣFy = Ry; F₁sinθ- 100 N = 800sin30° N


F₁sinθ = 400 N + 100 N
F₁sinθ = 500 N eq.1

ΣFx=Rx;F₁cosθ + 350 N = 800cos30° N


F₁cosθ= 692.82 N - 350 N
F₁cosθ = 342.82 N eq. 2

Equate eq. 1 and eq. 2:


F₁sinθ=500 N
F₁cosθ342.82N

sinθ=500 N
cosθ342.82N

tanθ =500 N
342.82N

θ=tan⁻¹500N
342.82 N

θ=55.56°

Substitute the θ to eq.1:


F₁sinθ= 500 N
F₁sin55.56°= 500 N
F₁ =500 N
sin55.56°

F₁ =606.27 N

α β
From the Figure:
F2 = FX2 + FY2 + FZ2

F =√(FX2 + FY2 + FZ2)

Where:
FX = component along x - direction
FY = component along y - direction
FZ = component along z – direction

d2 = a2 + b2 + c2
Where:
a = distance OA
b = distance OB
c = distance OC
d = distance OD

COORDINATION DIRECTION ANGLES:


cos2α + cos2 β + cos2 δ = 1
Where:
α= angle of F from x – axis
β = angle of F from y – axis
δ = angle of F from z – axis

RELATION BETWEEN FORCES AND DISTANCE


F/d = FX / a = FY /b = FZ/c
FX = F (a / d)
FY = F (b / d)
FZ = F (c / d)

SAMPLE PROBLEMS:
PROBLEM 1: The non-coplanar concurrent force system is shown. Determine the magnitude
and coordinate direction angles of the resultant force.

Required:
R, α, β, δ
Solution:
A = 12 kN, d = 114
12 kN / √114 = AX / 5 = AY / 8 = AZ / 5
AX = 5.62 kN
AY = 8.99 kN
AZ = - 5.62 kN

B = 25 Kn, d = 50
25 kN / √50 = BX / 3 = BY / 5 = BZ / 4
BX = -10.61 kN
BY = 17.68 Kn
BZ = -14.14 kN

C = 19 kN, d = 110
19 kN / √110 = CX / 5 = CY / 7 = CZ / 6
CX = - 9.06 kN
CY = - 12.68 kN
CZ = - 10.87 kN

COORDINATE DIRECTION ANGLES:


α = cos-1 (-14.05 / 97.77) = 104.98o
β = cos-1 (24 / 97.77) = 65.45o
δ = cos-1 (-50.65 / 97.77) = 151.21o

∑FX = 5.62 - 10.61 – 9.06 = -14.05 kN


∑FY = 10 + 8.99 +17.68 - 12.68 = 24 kN

FROM R =√ (FX2 + FY2 + FZ2)


R = 57.77 kN

PROBLEM 2: Three forces act on the hook. The resultant forces have the magnitude and
direction shown. Calculate the value of force F3 and its coordinate direction angle with respect to
Z – Axis.

Solution:
∑FX = RX
80 (d / 5) + F3X = (120 cos 45) (sin 30)
F3X = -21.57 N

∑FY = RY
F3Y = (120 cos 45) (cos 30)
F3Y = 73.48 N
∑FZ = RZ
80 (3 / 5) – 110 + FZ3 = 120 sin 45
F3Z = 146.85 N

From F3=√ (F3X2 + F3Y2 + F3Z2)


F3 = 165.62 N

δZ = cos -1 (146.85 / 165.67)


δZ = 27.54o
MOMENT OF A FORCE

It is the tendency of a force to cause rotation about a point of an axis.

In symbol: M=fd

Where:
M = moment
f = force (N, lb.)
d = is the perpendicular distance from the line of action of a force to a point of axis of
summation (m, ft)

M=fd

M(a)F= 25N (3m)


M(a)F = 75 N·m
SIGN CONVENTION:

CLOCKWISE ↻ = POSITIVE
COUNTERCLOCKWISE ↺ = NEGATIVE

SAMPLE PROBLEMS:

PROBLEM 1: For the coplanar non-concurrent forces shown, calculate the moment about
points A,B,C and D.

Required: MA, MB , MC , MD

Solution:

MA = -10 kN (5m) + 4kN (cos 40°) (5m) - 4kN (sin 40°) (1m) + 18kN (sin 20°) (2m) -18kN (cos
2 5 3 2
20°) (4m) + 6 kN (2m) + 15kN ( ) (2m) + 15kN ( ) (1m) - 8kN ( ) (1m) - 8kN ( )
√ 29 √ 29 √13 √ 13
(3m)
MA = -75.50 kN·m↺
MB = -10 kN (1m) + 4kN (cos 40°) (1m) + 4kN (sin 40°) (2m) + 18kN (sin 20°) (2m) - 6 kN
2 5 3
(1m) - 15kN ( ) (2m) + 15kN ( ) (4m) + 8kN ( ) (3m)
√29 √29 √13
MB = 44.43 kN·m↻

2
MC= -10 kN (6m) + 4kN (sin 40°) (3m) + 18kN (sin 20°) (3m) - 6 kN (2m) + 15kN ( ) (3m)
√ 29
5 3 2
+ 15kN ( ) (5m) - 8kN ( ) (2m) +8kN ( ) (1m)
√ 29 √13 √13
MC = 31.66kN·m↻

2
MD= - 4kN (sin 40°) (1m) - 18kN (sin 20°) (3m) -18kN (cos 20°) (4m) + 6 kN (2m) - 15kN (
√ 29
5 3 2
) (3m) + 15kN ( ) (1m) +8kN ( ) (4m) - 8kN ( ) (3m)
√ 29 √ 13 √13
MA = -66.17kN·m↺

VARIGNON’S THEOREM
The moment of the resultant force about an axis is equal to the sum of the moment of the
component forces about the same point or axis of rotation.

In symbols: MR = ∑MCOMP

SAMPLE PROBLEMS:

PROBLEM 1: For the coplanar parallel forces shown. Determine the magnitude and location of
the resultant forces.

Solution:

R = ∑FV
R = -200lb -350lb +280lb -230lb -120lb +140lb
R = 480 lb. ↓

M(a)R= M(a)COMP
480 (x̅) = 200lb (1’) +350lb (2’) -280lb (3’) +230 (5’) -140lb (6.5’) +120lb (7.25’)
(x̅) = 2.44’

PROBLEM 2: Find the values of P and F so that the four forces produce a downward resultant
of 350 N located 4 m. from the left end of the bar.
Given: 350 N
Required: P and F

Solution:

R = ∑FV
120N – P + F – 190N = -350N
P – F = 280N (equation 1)

M(a)R = M(a)COMP
350N (4m) = P (2m) – F (5m) + 190N (7m)
2P – 5F = 70N (equation 2)

Solving simultaneously:
P = 443.33N
F = 163.33N

PROBLEM 3: A force F passing through C produces a clockwise moment of 600 lb.-ft. about
A and a counter clockwise moment of 300 lb.-ft. about B. Determine the moment of F about O.

Given: M(a)F= +600 lb.-ft. ,M(b)F= -300 lb.-ft.


Required: M(o)F

Solution:

M(a)F = +600 lb.-ft.


Fy (3’) + Fx (4’) = +600 lb.-ft
3Fy + 4Fx = +600 lb.-ft (equation 1)

M(b)F = -300 lb.-ft.


-Fy (2’) -Fx (1’) = -300 lb.-ft
2Fy + Fx = 300 lb.-ft (equation 2)

Solving simultaneously:
Fy= 120lb
Fx= 60lb

RESULTANT OF ANY FORCE SYSTEM

SAMPLE PROBLEMS:
PROBLEM 1: The three forces shown on the grid produce a horizontal resultant through point
A. Find the magnitude and sense of P and F.

Solution:
Since the resultant is horizontal,
ƩFV= 0
4
100( ) – F = 0
5
F = 80 lb

M RA = ƩM component
A

3
R(0) = P(2’) + 100( )(2’) – 80 lb(1’)
5
P = 20 lb to the left

PROBLEM 2: The howe roof truss shown carries the given loads. Determine the value of
resultant force and its intersection with AB.

Required: R, x̄A
Solution:
R2 = (ƩFV)2 + (ƩFH)2
5
ƩFH = 440 N ( )
√61
ƩFH= 281.68 N

5
ƩFV = -440 N ( ) – 600 N
√ 61
ƩFV= 938.02

R = √ Ʃ FH Ʃ F ¿
2+ ¿
v
2

R = 979.40 N

−1Ʃ FV
Ɵ = tan
Ʃ FH
Ɵ = 73.29°

M RA =Ʃ M component
A

938.02 N (x̄)=300 N (3m) + 200 N (6m) + 100 N (9m) + 200N (


√61 m) + 120 N ( 61m)
2

x̄A= 5.03 m
PROBLEM 3: A flat plate is subjected to the coplanar system of forces shown. Determine the
resultant and its y and x intercept.

Required: R, xi, yi

Solution:
3 1
ƩFH= -290cos 60 – 360 ( ) + 450 ( )
√13 √5
ƩFH = -243.29 lb

2 2
ƩFV = 290sin 60 – 360 ( ) – 450 ( )
√ 13 √5
ƩFV = 351.04 lb

R = √ Ʃ FH Ʃ F ¿
2+ ¿
v
2

R = 427.11 lb

X-intercept
M OR = ƩM Ocomponent
3 2 2
351.04 lb (xi) = -290cos 60(3’) - 290sin 60(2’) – 360 ( )(3’) + 360 ( )(3’) + 450 ( )(4’)
√ 13 √ 13 √5
xi = 1.06

Y-intercept
M OR = ƩM Ocomponent
yi = 1.53
PROBLEM 4: In the rocker arm shown, the moment of F about O balances that of P about O.
Find F.

Given: P = 250 lb

Required: F

Solution:

M FO = M PO
2
F ( ) (6’’) = 250 lbcos 16.26 (5’’)
√5
F = 223.61 lb
F = 224 lb
EQUILIBRIUM OF FORCE SYSTEM

Equation of Static Equilibrium;


(A) ΣFH = 0
(B) ΣFV = 0
(c) ΣM0 = 0

TYPES OF SUPPORT SKETCH OF REACTING NUMBER OF REACTION


FORCES ELEMEMTS

FLEXIBLE CORD/
CABLE
SMOOTH SURFACE

ROLLER

ROCKER

SMOOTH PIN/ HINGE


FIXED

BALL-AND-SOCKET
SAMPLE PROBLEMS:

PROBLEM 1: For the truss loaded as shown, calculate the reactions at supports A and B.

REQ’D: RA, RB
SOLUTION:
ΣMB = 0;

RAy(62’) – 1200lb (20’) – 1500lb (38’) – 600lb ( 143 ) – 900 ( 23 × 14)– 300 ( √16305 )(38’) + 300
lb lb

7
( √ 305 )( 34 ×14) – 2500 ( √16305 ) ( 46 ' ) + 2500( √ 305
lb 7
) (7 ' ) - 2000 ( √16305 ) (54 ' )
lb
+

7
2000 (
√ 305 )
lb ( 3.5 ) = 0

RAy = 6104.83lb

ΣMA = 0;

2
( √ 305
2000lb
2 )+ 2500 ( √ 305 ) + 3000 ( 1.5 √ 305 ) + 1500 (24’) + 1200 (42’) – 900 ( × 14 ) –
lb lb lb lb
3
lb

14 5
600 ( ) - R (
√ 41 )
lb
B ( 62' ) = 0
3

RB = 4439.10lb
ΣFH = 0;

7
RAx + 7500lb ( √ 305 )– 1500 - R ( √441 )= 0
lb
B

RAx = 1266.94lb

RA = √ R Ax2 + R Ay2

RA = √ ¿ ¿

RA = 6234.91lb

FOR CHECKING:
ΣFV = 0;
16
RAy + RB = 2700 + 7500 ( √ 305 )
6104.83lb + 4439.10lb ( √541 )= 9571.18 lb

9571.18lb = 9571.18lb

PROBLEM 2: Three cylinders are piled in a rectangular ditch as shown. Assuming smooth
surfaces, determine the reaction between cylinder A and the vertical side of the ditch.

GIVEN:
WA = 16lb
WB = 32lb
Wc= 20lb
REQ’D: AH
¿ √ 112−7 2

= √ 72

SOLUTION:
Σ Fv=0;

RBC ( √1172 )– 20 = 0
lb

20(11)
RBC =
√ 72
RBC = 25.93lb

ΣFH = 0;

CH – RBC ( √772 )= 0
(23.93)(7)
CH =
11
CONSIDER CYLINDER B,
CH = 16.50lb
= √ 102−8 2
=6

ΣFV = 0

RBA ( 35 ) – 32 – R ( √1172 ) = 0
lb
BC

(25.93)( √72) 5
[
RBA = 32+
11 3 ]( )
RBA = 52.002 ( 53 ) 5
RBA = 86.67lb 3

4
ΣFH= 0;

RBC( 117 ) + R ( 45 ) – B = 0
BA H

25.93(7) 86.67(4)
+ = BH
11 5
BH = 85.84lb

CONSIDER CYLINDER A,
AV = 2Olb + 32lb + 16lb
AV = 68lb OR

ΣFV = 0;

AV – 16lb – RBA ( 45 ) = 0
86.67(3)
AV = 16 +
5
AV = 68lb

ΣFH = 0

AH – RBA ( 45 ) = 0
86.67(4)
AH =
5
lb
APROBLEM
H = 69.34 3: For the beam loaded as shown, calculate the reactions at the supports.

REQ’D: RA , RB
SOLUTION:
ΣMA = 0;
114kN (2m) + 144kN (3m) + 250kN ( 35 )(8 ) + 96
m kN
(13.5m)- RB(15m)= 0

RB = 210.40kN

ΣFH = 0;
RAx – 200kN = 0
RAx = 200Kn

ΣMB = 0;
RAy (15m) - 114kN (13m) – 144kN (12m) - 150kN (7m) – 96kN (1.5m) = 0
RAy = 293.60kN

RA = √ R Ax2 + R A y 2
RA = 355. 25kN

TO CHECK:

ΣFV = 0;
RAy + RB = 114kN+ 144kN + 150kN + 96kN
293.60kN +210.40kN = 504kN
504kN = 504kN

PROBLEM 4: A 12-ft bar of negligible weight rests in a horizontal position on the smooth
inclines. Compute the distance “x” at which load T= 100lb should be placed from point B to keep
the bar horizontal.

REQ’D: x
SOLUTION:
ΣFH = 0;

RA (sin30) – RB (sin40) = 0 Eq. 1

ΣFV = 0;

RA (cos30) + RB (cos40) – 200lb – 100lb = 0 Eq. 2

RA = 205.21lb
RB = 159.63lb
ΣMB = 0;
RA (cos30)(12) – 200lb(9) – 100lb (x’) = 0
X’ = 3.33’

USING TRIANGLE LAW:

300 RA
=
sin 110 ° sin 40°
RA = 205.21lb

300 RB
=
sin 110 ° sin 30 °
RB = 159. 63lb
ΣMB = 0;
RA cos30 (12’) – 200lb (9’) – 100lb (x’) = 0

205.21cos30 (12’) – 200(9) = 100(x’)


X’ = 3.33ft
PROBLEM 5: For the beams loaded as shown, calculate the support reactions.
REQ’D: RA, RB, RD
SOLUTION:
CONSIDER CD;

ΣFH = 0;

95kN( √529 ) – R Dx =0

RDx = 88.21kN

ΣMC = 0;

30kN (13m)- RDy(11m) + 95kN ( √229 )(9 ) + 12 (16


m kN/m
( 23 × 7.5−1.5) = 0
)(7.5m)

917.539
RDy =
11
RDy = 83.41kN

ΣMD = 0;

RC (11m) + 30kN (2m) – 95kN ( √229 )(2 ) - 12 (16


m kN/m 1
)(7.5m)(2m + 3m + + 7.5m ) = 0
3
RC = 41.87kN
RD = √ R Dx2+ R Dy2
RD = √ (88.21)2 +(83.41)2
RD = 121.40kN
FOR BEAM AB;
ΣFH = 0;
RAx = 0

ΣMA = 0;

RC = 41.87kN
7 1 14
10kN/m(7m)( – 2m)+10kN/m(7m)( )( – 2m) + RC(9m+1.5m) – RB(16m) = 0
2 2 3
637.968
RB =
16
RB = 39.87kN

ΣMB = 0;
7 7 7
RAy (16m) – 10Kn/m ( )( +11m) – 10kN/m(7)( + 11m) – RC(7m -1.5m) = 0
2 3 2
1711.952
RAy =
16
RAy = 107kN

FOR CHECKING:
1
RAy + RB = 10kN/m(7m) + 10kN/m( )(7m) + RC
2
39.87 + 107 = 105 + 41.87kN
kN kN kN

146.87kN = 146.87kN
PROBLEM 6: For the system of pulleys shown, determine the values of P to maintain
equilibrium. Assume that each pulley weighs 40 lb.

REQ’D: P
SOLUTION:
PULLEY NO.1 PULLEY NO.2
PULLEY NO. 3 PULLEY NO. 4

PULLEY NO. 5

W= 300 lb
AT PULLEY NO. 4 AT PULLEY NO. 2
ΣFV = 0;
ΣFV = 0;
3P1 – 40lb – 300lb = 0
3P1 = 340lb 3P – 40lb – P1 = 0
340 3P = 40 + 113.33
P1=
3 P= 51.11lb
P1= 113.33lb
PROBLEM 7: Find the distance “x” (measured along AB) at which a horizontal force of 60lb
should be applied to hold the uniform bar AB in the position shown. Bar AB is 10-ft long and
weighs 140lb. The incline and the floor are smooth.

REQ’D: x
SOLUTION:
ΣFH = 0;

60lb – RB ( √313 )= 0
60 √13
RB =
3
RB = 72.11lb

ΣFV = 0;
2
( )
RA + R B
√ 13
- 140lb = 0

2
RA = 140 – 72.11 ( )
√ 13
lb
RA = 100

ΣMB = 0;
RA (8’) – 140(4’) – 60(y’) = 0 x' y'
=
y’= 4’ 10' 6 '
40
x’ = 6 = 6.67’

x = 10’ – 6.67’ = 3.33 ft


PROBLEM 8: The cantilever beam is loaded as shown in the figure. Determine the reactions at
the fixed support.

Req’d: RA, MA
Solution:
ΣFH = 0;

RAx -145 kN ( 45 )=0


RAx = 1165 kN

ΣFv = 0;

RAy – 145 ( 35 ) - 14 kN/m (4m) - ( 12 ) (16 kN/m) (4m) = 0


RAy= 175 kN

RA = √ R Ax2 +R Ay2
RA = √ (1165)2 +(175)2
RA = 209.95kN

ΣMA = MA;
3 4
MA = 145
5 () 2 (
(0.75m) + 14 kN/m(4m) +2.25+ 0.75 ) ( 12 ) (16 kN/m) (4m)( 13 x 4+ 3) + 30 kN-
+

m
MA = 513.92 kN-m
PROBLEM 9: For the frame loaded as shown, determine the support reactions.

Req’d:
RA, RB
Solution:
ΣMA = 0;

- RB (6m) + 12kN/m (7m) (4.5m) + ( 12 ) (23 kN/m) (7m) (3.33m) -410 kN (2m) -280 ( 35 ) (4) +
4 1 1
280 ( ) (6tan 50) -( ) (20) (6/cos50) ( x (6 /cos 50) ) = 0
5 2 3
RB = 77.56 kN

ΣMB = 0;

RAx (6tan50) -12(7) (3.5 -2) - ( 12 ) (23)(7) ( 25 x 7−2) 410 (8) -280 ( 35 )(10) – (( 12 )(20)(6/cos50)
-

2 1 2
(cos50)) (12 -( ) x 6 ) + (( )(20)(6/cos50)(sin50))( x 6 tan 50 ) R (6) =0
+ Ay
3 2 3
RAy = 724.95 kN

TO CHECK:
ΣFv = 0;
RAy + RB = 410 +168 + 164.5 +60
802.50kN = 802.50 kN
PROBLEM 10: The uniform concrete slab has a weight of 5500lb. The slab is held in the
horizontal plane by three parallel supporting cables. Determine the tension of cables TA and TB.

Req’d: TA, TB

Solution:
Project loads on XZ plane
ΣMC = 0;
TB (6’) – 5500lb (3’) = 0
TB = 2750 lb

Project loads on YZ plane


ΣMC = 0;
TA (6’) - TB (3’) = 0
TA = 1375 lb
ANALYSIS OF STRUCTURES

A. TRUSS
i. By method of joints
ii. By method of sections
iii. By graphical method

B. FRAME
i. By method of members

ANALYSIS OF TRUSS
1. Method of Joints
Equations:
a. ΣFH = 0;
b. ΣFV = 0;

2. Method of Sections
Equations:
a. ΣFH = 0;
b. ΣFV = 0;
c. ΣMO = 0;

ASSUMPTIONS:
1. Members are straight between joints.
2. Loads are directly applied at the joints.
3. Members are fastened by smooth and frictionless pin or hinges.
4. Weight of members is negligible.
5. Lateral displacements or deformation are rejected.

METHOD OF JOINTS
SAMPLE PROBLEMS:
PROBLEM 1: For the truss loaded as shown in the figure, calculate the axial force in members
DC, DF, and EG. Use method of joints.

Req’d: PDG, PDF, PEG


Solution:
ΣFH = 0;
RAx = 23 kN

ΣMB = 0;
RAy (22m) -6(19m) - (7kN) (16m) -8kN (13m) -7kN ¿5m) +5kN ¿5m) -14kN ¿5m) +15kN ¿4.5m)
+10kN ¿3m) =0
RAy = 14.20 kN
Consider Joint A:
ΣFv = 0;

RAy – AB ( √15 ) = 0
AB = 31.75 kN (C)

ΣFH = 0;
1
-25kN + AC –AB
√5 ( )
=0

AC = 53.40 kN (T)
ΣFv = 0;
AB −6 kN = 0
AB = 6kN (T)
Consider Joint C:
ΣFH = 0;
CE - AC = 0
AC = 53.40 kN (T)
Consider Joint B:

ΣFH = 0;
2 2 2
31.75kN
√5 ( ) ( ) ( )
–BD
√5
-BC
√5
=0

BD + BE = 31.75 kN (1)

Consider Joint E: ΣFV = 0;


1
-BE ( )
√5
- 7kN + DE = 0

DE = 10kN

ΣFH = 0;

EG – CE + BE ( √25 )= 0
2
EG = 33.40 kN – 6.71 ( )
√5
Consider Joint D:
ΣFH = 0;

10kN + 25.04 ( √25 ) –DF ( √25 )- DG = 0


ΣFV = 0;
1 1
25.04 ( )
√5
-10kN –DF
√5( )+ DG
PROBLEM 2: Determine the axial force in members BE and BD for the truss loaded as shown
1
by method of joints.
( )
√2
=0

DF = 25.04 kN (C)
Req’d: BE and BD
Solution:
ΣMA = 0;
1
RA ( )
√ 13
(20 +( 4
Tan35 ) + RAY ( √213 ) (11’) – 2000lb (32 x 25' ) -1500lb (31 x 25' ) - 400lb (20’)
-600 (10’) + 800 (10) = 0
RA = 1447.30 kN

@Joint A: ΣFV = 0;
3
RA ( )
√ 13
- ABsin41.71 – ACsin35 =0

ΣFH = 0;
3
RA ( )
√ 13
- ABcos41.71 + ACcos35 =0

AB = 1486.74 lb
AC = 374.86 lb

ΣFH = 0;
@ Joint B
ABcos41.71 – 80 ( √661 ) + BE ( √229 ) = 0
ΣFV = 0;

ABsin41.71 – BC + 30 ( √561 ) - BE ( √329 ) = 0


BD = 2795.23 lb (C)
BE = 2793.48 lb (C)
@ Joint C
ΣFH = 0;
ACcos35 + BC -400 = 0
BC = 185 lb (T)

ΣFV = 0;
-ACcos35 + CE = 0
CE = 307.07 lb (T)

METHOD OF SECTIONS

SAMPLE PROBLEMS:

Problem 1: For the truss loaded as shown in the figure, calculate the axial force in member DF.
Use method of sections.
Solution:
Σ M D=0
From Section [1]-[1]:
14.20kN ( 6m ) +25 kN ( 5m ) −6 kN ( 3m ) −EG ( 3 m )=0
EG=4 7.46kN (T )

Σ M A =¿0
1
6 kN ( 3m ) +7 kN ( 6 m ) +10kN ( 3m ) −DG( )(3¿¿ m)−DG(1 /2)(6m )=0
√2
DG=14.14 kN (T )

Σ M G =0
2
−7 kN ( 3m ) −6kN ( 6m ) −14.20kN ( 9m ) +10 kN ( 3m ) −DF (
√5
)(3m )−DF (

Problem
2: The
structure
shown
consists of
two
simple
Σ F V =0
5

sections.
EF CF and−600
trusses joined by bars CF, BE, and DG. Find the force in bars BE,
√ 34 ( )
lb
DG. Use=0 method of

EF=699.17lb

Σ F H =0

FG+ EF ( √334 )=0


FG=4050lb (T )

From Section [1]-[1]:

Σ F H =0
3
Required:
BE−CF+ DG ( )
BE, CF, DG,
√ 34
=0 FG
Solution:
3 RG =1350 lb
3960lb −CF + 474.34lb
√ 34 ( )
=0 y

Consider Section [1] - [1]:


lb
CF=4420 (T ) Σ F V =0

1350lb −900 lb + DG ( √110 )=0


DG=474.34 lb ( T )

Σ M E=0

CF ( 5¿ )−600lb ( 3¿ ) −1350lb ( 12¿ ) −DG ( √110 ) ( 12 )− DG( √310 ) ( 5 ) =0


¿ ¿

CF=4410lb (T )
Σ M F =0
1 Σ M G =0
BE ( 5 ¿ ) +900lb ( 3¿ ) −1350lb ( 15¿ ) −DG ( )
√ 10
( 15¿ ) =0
−900lb ( 12¿ )−600lb ( 15¿ )+ BE ( 5¿ )=0
BE=396 0lb (C) From Joint F:
BE=3920lb
(C)
Problem 3: For the structure loaded as shown, determine the axial force in members BC, BF, &
C. Use method of sections.

Required:
BC, BF, CF

Solution:
Consider Section [1]-[1]:
Σ M J =0

35kN ( 8m ) +30 kN ( 4m ) −BC ( 257 ) (4 )−BC ( 2425 ) ( 8.5 )=0


m m

BC=43.10kN (T )

From Section [2]-[2]:


Σ M I =0

35kN ( 4 m )−BC ( 2425 )( 13 × 3.5+5 × 23 ×3.5)−CF ( √12265 )( 7.33 )=0


m

CF=−30.24 kn (C)

From Section [3]-[3]:


kN
R A =63.33
ANALYSIS OF FRAME: METHOD OF
y

Σ M G =0 MEMBERS EQUATIONS
(a) ∑FV = 0
24 2 7
(b) ∑FH = 0 ( )(
−63.33kN ( 6m ) + 25kN ( 4m ) −BF ( 4 m ) + BC
25 3 ) ( )
2.5+ × 3.5 + BC
25
(c) ∑MO = 0 BF=−7.93kN ( T )
SAMPLE PROBLEMS:

PROBLEM 1: For the structure loaded as shown, determine the horizontal and vertical
components of the pin reactions at b as they act upon member CD.

Req’: Bv, Bh
Solution:
Consider Ab;

∑ MA = 0

300lb (10’)+ BH(10’) – BV(20’) =0


2BV – BH = 300 LBS. (EQ.1)

CONSIDER MEMBER CD;

SOLVE THE EQUATION 1 AND 2


REACTIONS AS THEY ACT UPON MEMBER CD
∑MD = 0
BH = 216.67LB
BlbV =(25’)+
70 BHLB
258.33 (20’) – BV(10’) =0
2BH– BV = 175 LBS. (EQ.2)
ACT UPON AB
BH = 216.67LB
BV = 258.33LB
PROBLEM 2: The a-frame shown is supported by a hinge at A and a roller at e. Compute the
horizontal and vertical components of the hinge at B and C as they act upon member AC.

REQ D; BH, BV, CH, CV


Solution:

CONSIDER THE WHOLE STRUCTURE:


∑MA = 0
240” (11”) – RE (10”) = 0 RE = 264 lb.
∑FV = 0
RE – 240 lb – RAY = 0 RAY = 24 lb

CONSIDER MEMBER AC; CONSIDER THE HORIZONTAL


MEMBER

∑MD = 0
-BV (6’) + 290 (3’) = 0 BV = 120lb
BACK TO AC:
∑MC = 0
RAY = 24lb
LB
-24 (5’) + BV (3’) – BH(6’) = 0 BU = 40LB
∑FH = 0 CH = B H CH = 40LB
∑FV = 0
0 = BV – CV – 24 LB CV= 96LB
SUMMARY:

RAECTIONS UPON MEMBER AC:


BV = 120 lb
BH = 40 LB
CV = 96 lb
CH = 40 lb
PROBLEM 3: For the three-hinged arch loaded as shown, determine the support reactions at A
and B.

REQ D; RA, RB

SOLUTION:
CONSIDER THE WHOLE STRUCTURE:
∑Mc = 0
RAY (107 + 10cos25o) – RAX (10sin25o) – 25sin20o (10) – 30cos30o (10) – 92sin40o (10) = 0
19.06 RAY – 4.23 RAX = 936.68KN (EQ. 1)

Consider AC;
CH∑Mc = 0
RAY (10cos25o) - RAX (10+10sin25°) -25cos20°(10)
-30sin30°(10) = 0 EQ.2
Solve Equation 1 & 2
RAX = 4.94 KN RA = √ R AX2 + R AY 2
RAY = 50.24 KN RA = 50.48 KN

Consider the whole structure:


∑ M A =0
RAY (10+10cos20°) –RAX (10sin25°) +25sin20°(10cos25°) +25cos20°(10sin25°)
+30cos30°(10cos25°) 30sin30°(10sin25°) +92sin40°(10cos25°) +92cos30°(10sin25°) =0
19.06 RBY + 4.23 RBX = 713.75 KN EQ.3

M C =0 BC;
∑Consider
-RBY – RBX (10m) +RBX (10) +92cos40° (10m) =0
RBY – RBX = 92cos40° EQ.4

Solve Equations 3&4:


RBX = 27.03KN
RBY = 43.45 KN
CHECKING:
RBY+RBY?¿92 sin 40°+30cos30°+25sin20°
93.69KN¿ 93.67KN
RAX + RBX+25cos20°+30cos30°+25sin25°=92cos40°
70.46KN¿ 70.48KN
PROBLEM 4: For the frame show, determine the horizontal and vertical compensate of the
hinge force at B as it acts upon member AC.

REQ’D= BV, BH

Solution:
Consider the pulley:

∑ F V =0 ∑ F M =0
Av=600lb AH=600lb
Consider Member DE:

∑ M E=0
-600lb (16’) +BM (6’) =0
BH=1600lb

Consider AC:
∑ M C =0
600lb (12’)-600lb (16’)-BH (6’) 7Bv (8’) =0
BV=1500lb
Reactions as it act upon member AC:

BV=1500lb ( )

BH=1600lb ( )
PROBLEM 5: The frame shown is hinged to rigid
supports at D and E and roller supported
at A. Find the components of the
hinge force at D and E caused by the
given loads.
REQ’D: RAX, RAY, REX, REY

Solution:
Simplest member (to be stated) which is BA (caused by a roller support only)

CONSIDER MEMBER BA;

∑PH = 0
BH = 0
∑MA = 0
BV (8’) – 90lb (4’) = 0
BV = 45lb

∑FV = 0 ∑MC = 0

CONSIDER MEMBER CD; REY – CV – 150 = 0 0 = BV (4’) – RAY (9’)


R = 175lb RDY = 20lb
EY

∑MC = 0 ∑FV = 0
lb
0 = 150lb (6’) – REYC(12’)
V + RAY – 45 = 0
+ REX (6’)
R =200lb CV = 25lb
EX

∑FH = 0
REX = CH = 200lb
RDX = 200lb
∑FH = 0
CH – RDX = 0

PROBLEM 6: For the beam loaded as shown, determine the support reaction.

REQ’D: RA,RB, RD
Note: moment equation can only use once

SOLUTIONS:
From section 1-1;
∑ MC = 0
-RD (5m) + ½(38)(8)(2/38) = 0
RD = 162.13 KN
∑ FX = 0
RAX – 175(3/5) = 0
RAX = 105 KN

CONSIDER THE WHOLE STRUCTURE:


∑ MA = 0
18kN/m(10)(2) + 1/2(24kN/m)(10)(1/3*10m-3m) + 175(4/5)(7) - 104kN/m + 1/2 (38)(8)(2/3*8+10) –
RD(15m) – RB(6m) = 0
RB = 195.79 KN
∑ MB = 0
RAY(6) - 18(10)(4) - 1/2(24)(10)(4)(5.6) + 175(4/5)(1) - 104 + 1/2 (38)(8)(4+2/3*8) - RD(9) =
0
RAY= 234.08 KN
CHECKING:
∑ FV = 0
RAY + RB +RD = 300kN + 175(4/5) + 38(8)
234.08kN + 195.79kN + 162.13kN = 300kN + 140kN + 152kN
592kN = 592kN
RA = RAX2 + RAY2
RA = 1052 + 234.082
RA = 256.55kN

FRICTION
Consider a block resting on a rough surface and then subjected to load P;

F
θ
N
Friction – is the contact resistance exerted by a body upon another body when one of the bodies
tends to move past another body.
In symbol:
F = μN where: F = Frictional force (N, lb)
μ = Coefficient of static friction
N = Normal force (N, lb)
Factors affecting the value of friction:
1. Types of material
2. Roughness of the surface
3. Normal force developed between contacting surface

W P
R
F N

θ R F
N

F
tanθ=
N
F
μ=
N
∴ tan θ=μ

SAMPLE PROBLEMS:

PROBLEM 1: Determine the range of values of P so that the 250 N block will tend to move (a)
up the incline and (b) down the incline, use μ = 0.25.

30°
°
Given:
μ = 0.25
W = 250 N
Req’d:
Pmax
Pmin
Solution:
a. Up to the incline
P ΣFy = 0 ΣFx = 0
N – 250cos30° = 0 P – F – 250sin30° = 0
F N = 216.51 N P = 179.13 N

Direction of movement from F = μN


N F = 0.25(216.51 N)

b. Down the incline


250 N
P ΣFy = 0 ΣFx = 0
N – 250cos30° = 0 P + F – 250sin30° = 0
F N = 216.51 N P = 70.81 N
F = 54.13 N

PROBLEM 2: Bodies A & B are joined by a cord parallel to the inclined plane as shown.
Determine the angle θ at which motion impends. What is the tension in the cord?

A μ = 0.50

μ = 0.20
θ
WA = 200 lb
WB = 300 lb
Given:
WA = 200 lb
WB = 300 lb
Solution:
Consider Block A
200 lb
ΣFy = 0 ΣFx = 0
T NA – 200cosθ = 0 T – FA – 20sinθ = 0
NA = 200cosθ T = 200sinθ – 40cosθ 1
From F = μN
FA = 0.20(200cosθ)
FA = 40cosθ
NA
FA
Consider Block B
300 lb ΣFy = 0 ΣFx = 0
NB – 300cosθ = 0 -T + FB – 300sinθ = 0
NB = 300cos T = 150cosθ – 300sinθ 2
T From F = μN
FB = 0.50(300cosθ)
NB
FB FB = 150cosθ

Solve equations 1&2:


200sinθ – 40cosθ = 150cosθ – 300sinθ
500sinθ = 190cosθ
190
tanθ=
500

θ = 20.81°
T = 33.63 lb
PROBLEM 3: A horizontal bar, 10 ft long and of negligible weight, rests on rough inclined
planes as shown. If the angle of friction is 15, how close to B may the 200 lb force be applied
before motion impends?

Given:
θ = 15°
Req’d:
x =? FA
Solution: 25°
Tanθ = μ NB
μ = 15° NA 25° 60°
FB
FA = tan15°NA
FB = tan15°NB

ΣFH = 0
FAcos25° - NAsin25° + FBcos60° - NBsin60° = 0
tan15°NAcos25° + NAsin25°+ tan15°NBcos60° - NBsin60° = 0 1
ΣFV = 0
-FAsin25° + NAcos25° + FBsin60° + NBcos60° - 300 lb = 0
-tan15°NAsin25° + NAcos25°+ tan15°NBsin60° + NBcos60° = 300 2
NA = 205.69 lb; FA = 55.11 lb
NB = 186.98 lb; FB = 50.10 lb
ΣMB = 0
-200 lb(x) – 100(8) + NAcos25°(10) – FAsin25°(10) = 0
x = 4.8’

Using Triangle Law:


R
RA θ = 15°
40° N
95° 300 lb
45°

RB F
RA RB 300 lb
Tanθ =F/N cos15° = NA/RA
= =
sin 45 ° sin 40° sin 95 ° F = Μn NA = 205.69 lb
μ = F/N sin15° = FA/RA
RA = 212.94 lb sin15° = FB/RB FA = 55.11 lb
RB = 193.57 lb FB = 50.10 cos15° = NB/RB
NB = 186.98 lb
PROBLEM 4: Find the least value of P that will
just start the system of blocks shown moving to the right. The coefficient of friction under each
block is 0.30.

Given:
WA = 200 lb μ = 0.30
WB = 300 lb
Req’d: P =?
Consider Block A: Consider the Pulley:

T
ΣFy = 0
NA – 200 (4/5) = 0 3
200 lb
T
NA = 160 lb 4

From F = μN FA
F = 0.30(160 lb)
F = 48
ΣFx = 0 NA
T – FA – 200(3/5) = 0
T = 168 lb
Consider block B:
300 lb ΣFV = 0
P
T = 168 lb Pcosα – 90 lb + 0.30Psinα = 168 lb
α
P (cosα + 0.30sinα) = 258 lb
P = 258 lb / (cosα + 0.30sinα)
FB
ΣFV = 0 NB
*get the derivative of P*
dP 258(−sin α +0.30 cos α )
NB + Psinα – 300 lb = 0 =
dα (cos α +0.30 sin α )2
NB = 300 lb – Psinα sinα =0.30 cos α
ΣFH = 0 tanα =0.80
-168 lb + Pcosα – FB = 0, but FB = 0.30; α =16.70 °
FB = 0.30(300 lb – Psinα) P = 247.12 lb
PROBLEM 5: Block A supports a load W = 1000 lb and is to be raised by forcing the wedge B
under it. The angle of friction for all surfaces in contact is 15°. Determine the force P which is
necessary to start the wedge under the block. The block and wedge have negligible weight.

Given: θ = 15°
Req’d: P
Solution:

Consider block A:
W = 1000 lb ΣFH= 0
FA

NA

FAB 20°
FABcos20° + NAsin20° - NA = 0
But μ = tan15°
∴FA = tan15°NA
FAB = tan15°NAB
tan15°NABcos20° + NABsin20° - NA = 0 ––Eq. 1
ΣFV= 0
20°
-FABsin20° + NABcos20° - FA = 1000 lb
-tan15°NABsin20° + NABcos20° - tan15°NA = 1000 lb ––Eq. 2
NA = 861.92 lb FA = 230.95 lb
NAB = 1451.51 lb FAB = 388.93 lb

Consider the wedge:


NAB ΣFV= 0
20° -NABcos20° + FABsin20° + NB = 0
20°
NB = 1230.90 lb
FAB But FB = tan15°NB
P FB = 329.83 lb
ΣFH= 0
P – FB – FABcos20° - NABsin20° = 0
P = 1191.75 lb
FB

NB

PROBLEM 6: A homogeneous cylinder, 3 ft in diameter and weighing 400 lb is resting on two


rough inclined surfaces as shown. If the angle of friction is 16°, find the moment M applied to
the cylinder that will start it rotating clockwise.
M

1.5’

42°
55°
Given: W = 400 lb
Req’d: M
Solution:
tan16° = μ
FA = μNA
FA = tan16°NA
FB = tan16°NB
ΣFH = 0
FAcos42° + NAsin42° + FBcos55° - NBsin55° = 0
tan16°NAcos42° + NAsin42° + tan16°NB(cos55° - NAsin55°) = 0 ––Eq. 1
ΣFV = 0
-FAsin42° + NAcos42° + FBsin55° + NBcos55° - 400 lb = 0
-tan16°NAsin42° + NAcos42° + tan16°NBsin55° + NBcos55° = 400 lb ––Eq. 2

Solve equations 1 & 2:


NA = 243.79 lb FA = 69.91 lb
NB = 978.53 lb FB = 94.20 lb
ΣMO = 0
M – FB (1.5’) – FA (1.5’) = 0
M = 243.17 lb-ft
R
16° N
Using triangle law
400 lb RA RB
= =
RA
sin 83 sin 39 sin 58 F

58° RA = 253.62 lb cos16° = NA / RA


83° 400 lb RB = 341.77 lb NA = 243.80 lb
39° cos16° = NB / RB = 328.53 lb
RB

PROBLEM 7: Block A weighs 120 lb, block B weighs 200 lb and the cord is parallel to the
incline. If the coefficient of friction for all surfaces in contact is 0.25, determine the angle θ of
the incline for which motion of B impends.
Given:
WA = 120 lb
WB = 200 lb
μ = 0.25
Req’d: θ = ?

Solution:
Consider block B:
ΣFy = 0 ΣFx = 0
NA NB – NA – 200cosθ = 0 -FA – FB + 200sinθ = 0
NB = 320cosθ 200sinθ = 30cosθ + 80cosθ
FA
FB = 0.25 NB tanθ = 110/200
FB = 0.25(320cosθ) θ = 28.81
FB = 80cosθ
NB
FB

Consider block A:
ΣFy = 0 ΣFx = 0
NA – 120cosθ = 0 -T + FA + 120sinθ = 0
NA = 120cosθ T = 30cosθ + 120sinθ

But FA = μNA T = 30cos (28.81) + 120sin (28.81)


FA FA = 0.25(120cosθ) T = 84.12 lb
FA = 30cosθ
NA
BELT FRICTION

Consider a pulley with tensile forces T1&T2:

B B

Relationship between T1&T2:


T 2 µB
=e ; T 2=T 1 e µB
T1
Where:
T2= larger tension (N, lb)
T1= smaller tension (N, lb)
µ= coefficient of belt friction
B= angle of contact between the belt and pulley (rad)
If θ= 40°
Then;
π
B= 130°X
180°

If θ= 40°
Then;
π
B= 40°X
180°

SAMPLE PROBLEMS:

PROBLEM 1: A torque of 240 lb-ft acts on the brake drum shown. If the brake band is in
contact with the brake drum through 250° and the coefficient of friction is 0.30, determine the
force P at the end of the brake lever.
GIVEN:
T=240 lb-ft
µ= 0.30
B= 250°

REQUIRED: P

SOLUTION:
Consider the brake drum:
Σ M o= 0
240 lb-ft x 12 } over {{1} ^ {'}¿ - T2(8”) + T1(8”)= 0
T2 - T1 = 360 lb 1

FROM T 2=T 1 e µB

π
(0 .30 )(250° x ¿ )¿
T2 = T1 180° 2
e

SOLVE T1 AND T2 FROM EQUATION 1 AND 2


T1 = 133.21 lb ; T2 = 493.21 lb

Consider the brake lever:


ΣMA = 0
T2 (2”) – P (16”) = 0
493.21 lb (2”) – P (16”) = 0

493.21 lb(2 ”)
P=
16 ”
P= 61.65 lb
PROBLEM 2: For the system of blocks shown, determine the weight of block B so that it will
start to move downward.

µ= 0.25 GIVEN:
W A= 250 N
REQUIRED: WB
µ= 0.30
3

SOLUTION:
Consider the FBD of block A:

Σ Fy = 0 Σ Fx = 0
4 3
N- 250( ) = 0 -F + T1 – 250 ( ) = 0
3 5
N = 200 N T1 = 200 N

FROM F = µN
F= 0.30 (200 N)
F = 60 N

Consider the Pulley:

126.87° FROM T 2=T 1 e µB

θ = 0.25
µ
T1
π
B = 126.87 x
180°
T 2=200 e(0.25)¿¿)
T2
T 2 = 365.29 N

Consider block B:
T2
Σ Fy = 0
WB = T2= 365.29 N

WB

PROBLEM 3: The coefficient of friction is 0.20 between the rope and the fixed drum and
between all surfaces in contact. Determine the minimum weight w to prevent downward motion
of 1000 lb body.

GIVEN:

µ= 0.20
REQUIRED: w

Consider block w:
Σ Fy = 0 Σ Fx = 0
4 3
N1 - W =0 T1 -F1 - w =0
5 5
N1 = 0.80w T1 = 0.76 w
FROM F1 = µN
F1 = 0.20(0.80w)
F1 = 0.16w

Consider fixed drum:

FROM T 2=T 1 e µB

T2 = T1
e 0 .20 π
T1
T2 = 1.42 w
e 0 .20 π

T2

Consider the 1000 lb block:

Σ Fy = 0 Σ Fx = 0
4 4
N2 – N1 – 1000( ) = 0 F1 + F2 + T2 – 1000( ) = 0
5 5
N2 = 0.80w + 800lb 0.16w + F2 + 1.42w – 600lb = 0

FROM F2 = µN F2 = 600lb – 1.58 w 2

F2 = 0.20 N2 1
SOLVE FOR W USING EQUATION 1 AND 2
0.20[ 0.80w + 800lb] = 600lb – 1.58w
W= 252.87 lb

CENTROIDS AND CENTERS OF GRAVITY


Locations of Centroids for regular shape are:

1.) Rectangle 2.) Circle

3.) Triangle 4.)


Semi-Circle
5.) Quarter of a Circle

SAMPLE PROBLEMS:

PROBLEM 1: For the composite area shown, locate the centroids with respect to reference x
and y axes. All labels should be in mm.

Solution:
A1= 440mm (225mm) = 112200mm2
1
A2 = (110) (115) = 6325 mm2
2
π (90)2
A3 = = 12723.45 mm2
2
π (120)2
A4 = = 11309.73 mm2
4
1
A5 = (70) (60) = 2100 mm2
2
440 1 4 ( 120 ) 1
AT (x́) = A1 ( ) - A2 ( * 110mm) - A3 (90 * 80mm) – A4 {440 – } - A5 (440 - * 70)
2 3 3π 3
x́ = 213.36 mm
255 1 4(90) 4 (120) 1
AT ( ý) = A1 (
2
) - A2 (255 - * 115) - A3
3 [3π ] [
- A4 255−
3π ]
- A5 ( * 60)
3
ý = 126.65 mm

PROBLEM 2: Locate x́and ý for the composite area shaded as shown.All labels should be in
inches.

Solution:
AT =A1 -A2 -A3 -A4 -A5 +A6 = 297.80 in2
A1 = 23” (19”) = 437 in2
π (5)2
A2= = 39.27 in2
2
A3= 7(6.5) = 45.5 in2
1
A4= (12) (8.5) = 51 in2
2
A5 = (4)2 = 16 in2
π (4)2
A6= = 12.57 in2
4
23 4∗5 7 2 4 −4∗4
AT (x́) = A1 ( ) –A2 ( ) – A3 (16+ ) – A4 (11 + * 12) – A5 (11 - ) + A6 (7 + )
2 3π 2 3 2 3π
x́ = 10.25”
13 615 2
AT ( ý) = A1 ( ) – A2 (3+5”) – A3 ( ) – A4 (10.5+ * 8.5)
2 2 3
ý= 9.46”

PROBLEM 3: A homogeneous slender wire is bent as shown. Locate the center of gravity from
reference axes

Solution:
x
cos30o =
12.5
∴ x = 12.5 cos30o
LT = L1 +L2 +L3 +L4
LT =19” + 12” + (π) (10”) +25” = 87.42”
LT (x́) = L1(-22”)+ L2(-16”) + L3(0) + L4(12.5 cos30 + 10”)
x́ = -1.02”
19 2∗10
LT ( ý) = L1( )+ L2(0) + L3( ) + L4(12.5 sin30)
2 π
ý = 6.14”

PROBLEM 4: The semi-circular port of the line lies in the xy plane. Determine the location of
the centroid of the wire from the reference xy, yz and xz plane.

Solution:
L1 = 100mm
L2 = √ (100)2 +(160)2 = 188.68mm
L3 = π (120) = 377mm
LT = L1+ L2+ L3 = 665.68mm
LT (x́) = = L1 (0) + L2 (0) + L3 (120)
x́ = 67.69mm
160 2(120)
LT ( ý) = = L1 (0) + L2 (
2 [
) + L3
π ]
ý = 65.94mm
100 100
LT (ź) = L1 ( ) + L2 ( ) + L3 (0)
2 2
ź = 21.68mm

MOMENT OF INERTIA
(Area moment of inertia)

This is also called as second moment of area.

Moment of Inertia of Regular Shape Areas:

1. Rectangle

Ix c= bh³/12
Ix = bh³/3

Iy x= hb³/12
Iy = hb³/3
2. Circle

Ixc = Iyc = πr4/4

3. Triangle

Ixc = bh³/36
Ix = bh³/12

Iyc = hb³/36
Iy = hb/12

4. Semi - Circle

Ixc = 0.11r4
Ix = Iyc = πr4/8

5. Quarter of a Circle

Ixc = Iyc = 0.55r4


Ix = Iy = πr4/16
For composite areas, use the Transfer Formula
(Parallel area theory)

Ix = Ixc + Ad²
Iy = Iyc + Ad²

Where:
Ix = moment of inertia taken about any axis (mm4, in4)
Ixc = moment of inertia about centroidal axis (mm4, in4)
A = area (mm², in²)
d = distance from the centroid to the axis where moment of inertia is to be taken (mm, in)

SAMPLE PROBLEMS:

PROBLEM 1: For the composite area shown, calculate the moment of inertia about x, x1, y, and
y1 axes.
Req'd: Ix, Ix1, Iy, Iy1

Solution:

A1 = 205 (125) = 256.25 mm²


A2 = 1/2 (50) (55) = 1375 mm²
A3 = 125 (30) = 3750 mm²
A4 = π (60)²/4 = 1827.43 mm²

Ix = I 1 - I 2 - I 3 - I 4
I1 = 205(125)³/3 = 133.46x106 mm4
I2 = 205(125)³/36 + A2 (70 + 2/3 x 55)² = 15.88x106 mm4
I3 = 125(30)³/ 3 = 1.125x106 mm4
I4 = 0.055(60)4 + A4 [125 + 4(60)/3π]² = 28.72x106 mm4
Ix = 87.735x106 mm4

Ix1 = I1 - I2- I3- I4


I1 = 205(173)2/12 + A1 [(125/2) - 30]² = 60.43x106mm4
I2 = 50(55)³/36 + A2 [125 - (1/3)(55) - 30]² = 8.31x106mm4
I3 = 125(30)³/3 = 1.125 x106mm4
I4 = 0.055(60)4+ A4 [125- 30 -4(60)/3π]² = 14.38x106mm4
Ix1 = 36.62 x106mm4

Ix2 =I1-I2-I3-I4
I1= 205(125)³/3 = 133.46x106mm4
I2= 50(55)³/12 = 0.693x106mm4
I3= 125(30)³/12 + A3 (125 - 30/2)² = 45.656 x106mm4
I4= π (60) 4/16 = 2.545x106mm4
Ix2 = 84.566x106mm4

Iy = I1-I2-I3-I4
I1 = 125(205)³/3 = 358.96x106mm4
I2 = 55(50)³/12 = 0.57x106mm4
I3 = 30(125)³/12 + A3 (80 + 125/2)² = 81.03 x106mm4
I4 = 0.055(60)4+ A4 [205-4(60)/3π]² = 91.85x106mm4
Iy = 185.51x106mm4

Iy1 = I1-I2-I3-I4
I1= 125(205)³/12 + A1 (145 - 205/2)² = 133.425x106mm4
I2= 55(50)³/36 + A2 [(2/3)50 + 95???]² = 22.836x106mm4
I3= 30(125)³/12 + A3 [(125/2) - 60]² = 4.906 x106mm4
I4=0.055(60)4+A4 [60-4(60)/3π] ² = 4.085x106mm4
Iy1 = 101.598x106mm4

Iy2 = I1-I2-I3-I4
I1= 125(205)³/3 = 358.964x106mm4
I2= 55(50)³/36 + A2 [155 + (2/3)(50)]² = 48.961x106mm4
I3= 30(125)³/3 = 24.300x106mm4
I4= π (60)4/16 = 25.447x106mm4
Iy2 = 210.256x106mm4

PROBLEM 2: For the composite area shown, calculate the centroidal moment of inertia (Ixc, Iyc).

Req'd: Ixc, Iyc

Solution:

A = A1 - A2 - A3 - A4
A1 = 10.5(14) = 147 in²
A2 = π (3)²/2 = 14.14 in²
A3 = π (2)²/2 = 6.28 in²
A4 = 1/2 (4.5)(8.5) = 19.125 in2
A = 107.46 in²

Locate the centroid:

AT (x̅) = A1 ( 10.52 ) A ( 43(3)π ) - A (7) - A (6+ 10.5(4.5)


- 2 3 4
2 )
(x̅)= 5.003 = 5”

AT (y) = A1 ( 142 ) A ( 4) - A (14− 43(2)π ) - A ( 13 x 8.5)


- 2 3 4

(y)= 7.78 = 8”

Ixc =I1-I2-I3-I4
Ixc = 1528.90 in4
10.5 (14 )3
I1= ( 2 ) + A1 (y-y) 2 = 2490.43 in4

π (3)4
I2= ( ) A (y-y) = 233.85 in
8
+ 2
2 4

2
4x
I 0.11(3) + A (14− y −
3π )
2 4
3= = 182.93 in
3

3
4.5(8.5) 1
I =( ) 2
4 A ( y− (8.5 )) = 544.75 in
+ 4
4
36 3

Iyc = I1-I2-I3-I4
Iyc = 795.53 in4

10.5
I1= ¿ + A1 ( −x̅ ) 2 = 1359.75 in4
2
4 (3) 2
I2= 0.11(3)4 + A2 (x́− ) = 205.30 in4

π (3)4
I3= (8 )
+ A3 ( 7−x́ ) = 31.40 in4

4.53 ( 8.5) 1
I4 = (
36 ) 2
+ A4 (10.5−x− (4.5 )) = 377.52 in
3
4

You might also like